How Do You Calculate Force in Equilibrium Using Calculus?

  • Thread starter Thread starter BigFlorida
  • Start date Start date
  • Tags Tags
    Physics
Click For Summary
The discussion focuses on calculating the force required to hold a weight in equilibrium using calculus, specifically through the relationships T cos θ = W and T sin θ = F. The first part of the problem involves expressing F/W as a power series of θ, which was solved using the tangent function. The second part requires expressing F/W as a power series of x/l, but the user struggles with this transformation and considers different approaches, including using binomial expansion. Ultimately, the user finds a solution for the first part but still seeks clarification on the second part of the problem. The conversation highlights the challenges of applying calculus to equilibrium problems in physics.
BigFlorida
Messages
41
Reaction score
1

Homework Statement




  • The figure shows a heavy weight suspended by a cable and pulled to one side by a force F. We want to know how much force F is required to hold the weight in equilibrium at a given distance x to one side (say to place a cornerstone correctly). From elementary physics, T cos θ = W , and T sin θ = F .
    1. (a) Find F/W as a series of powers of θ.
    2. (b) Usually in a problem like this, what we know is not θ, but x

      and l in the diagram. Find F/W as a series of powers of x/l.

    Homework Equations

    [/b]

    Tcos(theta) = W
    Tsin(theta) = F
    sin(theta) = (x/l)
    cos(theta) = sqrt(l^2-x^2) = sqrt(1-(x/l)^2)[/B]

    The Attempt at a Solution



    The first part was a relatively simple solve, it was just a power series of the tangent function.

    The second part is what I am stuck on.

    I have (F/W) = (x/l)(1/sqrt(1-(x/l)^2)), but I have no clue how to turn the right side of this equation into a power series. Should I just create 2 different functions (one where x is a function of l and another where l is a function of x) and write 2 power series, or is there a way that I am not seeing? I have also tried just letting theta = arcsin(x/l) and substituting that into the power series I found in part 1, but that solution was incorrect. I am at a complete loss. Any help would be much appreciated.

    Thanks in advance!
 
Last edited by a moderator:
Physics news on Phys.org
Never mind, I solved it. It was a simple binomial expansion that I was overlooking.
 
I am still struck at the second part. can you please explain how you have solved it.
Thanks in advance
 
  • Like
Likes Ozymandias_II
Question: A clock's minute hand has length 4 and its hour hand has length 3. What is the distance between the tips at the moment when it is increasing most rapidly?(Putnam Exam Question) Answer: Making assumption that both the hands moves at constant angular velocities, the answer is ## \sqrt{7} .## But don't you think this assumption is somewhat doubtful and wrong?

Similar threads

  • · Replies 14 ·
Replies
14
Views
2K
Replies
4
Views
2K
  • · Replies 3 ·
Replies
3
Views
2K
  • · Replies 21 ·
Replies
21
Views
2K
  • · Replies 3 ·
Replies
3
Views
2K
  • · Replies 6 ·
Replies
6
Views
2K
Replies
3
Views
2K
  • · Replies 5 ·
Replies
5
Views
2K
Replies
4
Views
2K
  • · Replies 11 ·
Replies
11
Views
2K